Đến nội dung

Hình ảnh

USAMO 2017 ngày 2

usamo 2017

  • Please log in to reply
Chủ đề này có 2 trả lời

#1
Mr Cooper

Mr Cooper

    Sĩ quan

  • Thành viên
  • 496 Bài viết

$\boxed{\text{Bài Toán 4}}$. Cho $P_1,P_2,\dots ,P_{2n}$ là $2n$ điểm phân biệt trên đường tròn đơn vị $x^2+y^2=1$ và khác $(1,0)$. Mỗi điểm được tô đỏ hoặc xanh với đúng $n$ điểm đỏ và $n$ điểm xanh. Gọi $R_1,R_2,\dots, R_n$ là bất kỳ cách đánh dấu các điểm đỏ nào. Gọi $B_1$ là điểm xanh gần nhất với $R_1$ khi đi quanh đường tròn theo chiều ngược kim đồng hồ, xuất phát từ $R_1$. Sau đó gọi $B_2$ là điểm xanh gần nhất trong các điểm còn lại với $R_2$ khi đi quanh đường tròn theo chiều ngược kim đồng hồ, xuất phát từ $R_2$. Cứ như thế cho đến khi tất cả các điểm đều được đánh dấu $B_1, B_2,\dots ,B_{n}$. Chứng minh rằng số cung ngược chiều kim đồng hồ có dạng $R_iB_i$ mà chứa điểm $(1,0)$ thì không phụ thuộc với cách chọn thứ tự các điểm đỏ $R_1,R_2,\dots R_n$.

 

$\boxed{\text{Bài Toán 5}}$. Gọi $\mathbb{Z}$ là tập tất cả các số nguyên dương. Tìm  tất cả các số thực dương $c$ sao cho tồn tại một cách đánh dấu các điểm nguyên $(x,y)\in\mathbb{Z}^2$ bằng các số nguyên dương mà:

$\bullet$ Chỉ có hữu hạn số nguyên dương được dùng để đánh dấu

$\bullet$ Với mỗi số nguyên dương $i$ được dùng để đánh dấu, khoảng cách giữa hai điểm cùng được đánh dấu $i$ ít nhất là $c^i$.

 

$\boxed{\text{Bài Toán 6}}$. Tìm giá trị nhỏ nhất của biểu thức

$$\frac{a}{b^3+4}+\frac{b}{c^3+4}+\frac{c}{d^3+4}+\frac{d}{a^3+4}$$

trong đó $a,b,c,d$ là các số thực không âm và $a+b+c+d=4$.

 

 


Bài viết đã được chỉnh sửa nội dung bởi Mr Cooper: 24-04-2017 - 11:03


#2
Mr Cooper

Mr Cooper

    Sĩ quan

  • Thành viên
  • 496 Bài viết

$\boxed{6}$

Dễ thấy rằng: $\dfrac{1}{x^3+4}=\dfrac{3-x}{12}+\dfrac{x(x-2)^2(x+1)}{12(x^3+4)}$
$\Rightarrow \sum \dfrac{a}{b^3+4}\ge \dfrac{3(a+b+c+d)-(a+c)(b+d)}{12}\ge \dfrac{2}{3}$
Sử dụng bất đẳng thức $\text{AM-GM}$ ta có:$(a+c)(b+d)\le \dfrac{(a+b+c+d)^2}{4}=4$
Hoàn tất chứng minh



#3
Drago

Drago

    Sĩ quan

  • Thành viên
  • 462 Bài viết

$\boxed{6}$

Dễ thấy rằng: $\dfrac{1}{x^3+4}=\dfrac{3-x}{12}+\dfrac{x(x-2)^2(x+1)}{12(x^3+4)}$
$\Rightarrow \sum \dfrac{a}{b^3+4}\ge \dfrac{3(a+b+c+d)-(a+c)(b+d)}{12}\ge \dfrac{2}{3}$
Sử dụng bất đẳng thức $\text{AM-GM}$ ta có:$(a+c)(b+d)\le \dfrac{(a+b+c+d)^2}{4}=4$
Hoàn tất chứng minh

Biểu thức gạch chân ảo quá


$\mathbb{VTL}$






Được gắn nhãn với một hoặc nhiều trong số những từ khóa sau: usamo, 2017

1 người đang xem chủ đề

0 thành viên, 1 khách, 0 thành viên ẩn danh